Particles q1 = +8.0 UC, 92 = +3.5 uc, and

q3 = -2.5 uC are in a line. Particles qi and q2 are

separated by 0.10 m and particles q2 and q3 are

separated by 0.15 m. What is the net force on

particle q2?

Answers

Answer 1

Answer:

-22.3

Explanation:

Acellus

Answer 2

The net force on particle q2 is 21.7 N.

The given parameters:

Charge on particle 1, q1 = 8 μCCharge on particle 2, q2 = 3.5 μCCharge on particle 3, q3 = -2.5 μCDistance between particle 1 and particle 2 = 0.1 mDistance between particle 2 and particle 3, = 0.15 m

The net force on particle q2 is calculated as follows;

[tex]Q_2_{net} = q_{12} \ + \ q_{23}\\\\Q_2_{net} = \frac{kq_1q_2}{r_{12} ^2} \ + \ \frac{kq_2q_3}{r_{23}^2} \\\\Q_2_{net} = \frac{9\times 10^9 \times 8 \times 10^{-6} \times 3.5 \times 10^{-6} }{0.1^2} \ + \ \frac{9 \times 10^9 b\times 3.5 \times 10^{-6} \times (-2.5) \times 10^{-6} }{0.15^2} \\\\Q_2_{net} = 25.2 \ N \ - \ 3.5 \ N\\\\Q_2_{net} = 21.7 \ N[/tex]

Thus, the net force on particle q2 is 21.7 N.

Learn more about Coulomb's law here: https://brainly.com/question/24743340


Related Questions



The vector matrix [6 -2] is rotated at different angles. Match the angles of rotation with the vector matrices they produce.

Answers

Answer:

Hello your question is incomplete hence I will give you a general answer as regards rotation of vector matrix assuming angle of rotation  = [tex]\frac{\pi }{4}[/tex]

answer :

angle of rotation = [tex]\frac{\pi }{4}[/tex]

vector matrix produced = attached below

Explanation:

lets assume the vector matrix [ 6, -2 ] is rotated clockwise by an angle of [tex]\frac{\pi }{4}[/tex]

The resultant matrix = attached below

Result of the rotation =  attached below

attached below is the detailed solution

Answer: I got it right on Edmentum

Explanation: Answers attached below

Can anyone help please ​

Answers

Answer:

Option C, the current is constant through the wire.

Explanation:

Ok, so we know that as larger cross-section we have, smaller resistance, so makes sense to think that the current will be larger in the first piece of wire.

But now let's think in a simpler case.

Suppose that in the image we see pipes, and through these pipes, there is water flowing.

You would say that the water flow in WX is larger than the current in XY?

No, it can´t be, the volume of water that leaves WX in a lapse of time Δt, must be the exact same volume of water that enters XY in that lapse of time, then the water flow is constant.

The only thing that can happen is that the velocity at which the water moves in WX is smaller than the velocity at which the water moves in the other parts.

The same thing happens here, the current will be constant, and the thing that changes is the velocity at which the electrons flow in each part of the wire.

So if you actually had a wire like this, you would see that the thinner parts heat a lot more than the others, and this happens because the electrons in those parts have a lot more kinetic energy.

The correct graph is C.

528 nm light passes through a single slit. the second (m=2) diffraction minimum occurs at an angle of 3.48 degrees. what is the width of the slit

Answers

Answer:

1.74

Explanation:

right on acellus

Answer:

1.74

Explanation:

acellus

is ice a fluid at room temperature?

Answers

Answer:

Ice is the solid state of water, a normally liquid substance that freezes to the solid state at temperatures of 0 °C (32 °F) or lower and expands to the gaseous state at temperatures of 100 °C (212 °F) or higher.

Explanation:

A star that is one of the coolest,
about 3,200°C, is going to be which
of the following colors?
Resourcos
A. greenish
Help
B. bluish
C. yellowish
D. reddish

Answers

I believe it would be red

What 2 environments once existed where the Grand Canyon is located today?

Answers

The entire park area is considered to be a semi-arid desert, but distinct habitats are located at different elevations along the 8,000-foot elevation gradient. Near the Colorado River, riparian vegetation and sandy beaches prevail.

5. During a heat transfer investigation, students filled two cups with water. One cup contains 100mL of cold water while the other contains 100 mL of hot water.
A metal heat wire is placed inside the water of both cups and thermometers are attached to each cup. Which of the following BEST explains how the heat is transferred? Cold Water Hot Water.

A. This investigation would not result in any heat being transferred.

B. Convection, because the metal wire was heated through convection currents.

C. Radiation, because the metal wire was heated by electromagnetic waves.

D. Conduction, because the metal wire was in direct contact with water in both containers.​

Answers

Answer:

D.Conduction, because the metal wire was in direct contact with water in both containers

Conduction, because the metal wire was in direct contact with water in both containers.​ Option D is correct.

Given that,
Students filled two cups with water. One cup contains 100mL of cold water while the other has 100 mL of hot water. A metal heat wire is placed inside the water of both cups and thermometers are attached to each cup.
It is to be identified, the system contains which type of heat transfer between the cups

What is heat transfer?

Heat transfer is the process, in a thermodynamic system, when an object at a higher temperature is placed with an object at has a lower temperature, after some time the system attains equilibrium because heat flows from higher temperature to lower temperature.


In the given question,
Cups, that are connected through a metal wire, one is hot and the other is cold, Principal of conduction state that when there is a system of connected bodies the heat flows throughout the connected system till the thermal equilibrium attain. Implies heat flow from hot cup to cold cup through the wire through conduction heat transfer.

Thus, Conduction, because the metal wire was in direct contact with water in both containers.​ Option D is correct.

Learn more about heat transfer here:
https://brainly.com/question/20699475

#SPJ2



5.
What is gravity?
a) A force
D b) Energy
c) A push
d) All of them
science​

Answers

A). force

hope it helps and your day will be full of happiness

If a gas turned into a solid without going through the liquid state and how do you reverse it?

Answers

Answer:

put it in a volcano

Explanation:

Deposition is the phase transition in which gas transforms into solid without passing through the liquid phase. Deposition is a thermodynamic process. The reverse of deposition is sublimation and hence sometimes deposition is called desublimation.

Archerfish are tropical fish that hunt by shooting drops of water from their mouths at insects above the water’s surface to knock them into the water, where the fish can eat them. A 65-g fish at rest just at the surface of the water can expel a 0.30-g drop of water in a short burst of 5.0 ms. High-speed measurements show that the water has a speed of 2.5 m/s just after the archerfish expels it. What is the average force the fish exerts on the drop of water?
(a) 0.00015 N;
(b) 0.00075 N;
(c) 0.075 N;
(d) 0.15 N.

Answers

Answer:

(d) 0.15 N

Explanation:

mass of the fish, m₁ = 65 g = 0.065 kg

initial velocity of the fish, u₁ = 0

mass of water expelled by the fish, m₂ = 0.30 g = 0.0003 kg

time during which the water was expelled, t = 5.0 ms = 5.0 x 10⁻³ s

velocity of the water, v = 2.5 m/s

The magnitude of force of the exerted water is equal to the magnitude of force the fish exerted on the water.

The magnitude of force of the exerted water is calculated as follows;

[tex]F = ma = m\frac{v}{t} \\\\F = \frac{mv}{t} \\\\F = \frac{m_2 \ \times \ v}{t} \\\\F = \frac{0.0003 \ \times \ 2.5}{5 \times 10^{-3}} \\\\F = 0.15 \ N[/tex]

The correct option is D.

Therefore, the average force the fish exerts on the drop of water is 0.15 N.

what is energy and types of energy

Answers

Energy- the ability to do work/how things can change and move

Types
Potential Energy
Kinetic Energy
Nuclear Energy
Mechanical Energy
Sound Energy
Heat

write formula of Plank's constant ,angular momentum, impulse ,stress,Young s modulus.​

Answers

Answer:

E = hf. Plank's constant

L=mvr  Angular momentum

F • t = m • Δ v. Implulse

: σ = F/A  Stress

Explanation:

for youngs modulus its in the pictuure

I WILL REPORT YOU IF YOU ANSWER WITH LINK!!! WILL GIVE BRAINLIEST

You have two identical bowling balls that are 1.00 m apart as measured from their centers. There is an attractive force of gravity between them due to their mass. When a net charge of +0.40 nC is placed on each bowling ball, the force exerted by the electrostatic forces exactly balances the force of gravity on the bowling balls resulting in a net charge of zero. The formula for the electrostatic force is similar to the formula for the gravitational force. There is a constant that is multiplied by the magnitude of each of the two charges and divided by the square of the distance between them. What two conclusions can you draw from this information?


CHOOSE 2 ANSWERS

Answers

Answer:

I think is is

Explanation:

B and C why because i have a gut feeling

You desire to observe details of the Statue of Freedom, the sculpture by Thomas Crawford that is the crowning feature of the dome of the United States Capitol in Washington, D.C. For this purpose, you construct a refracting telescope, using as its objective a lens with focal length 86.3 cm. In order to acheive an angular magnification of magnitude 5.01, what focal length fe should the eyepiece have?

Answers

Answer:

the focal length of the eyepiece is 17.23 cm

Explanation:

The computation of the focal length of the eyepiece is shown below:

= Focal length of objective lens ÷ angular magnification magnitude

= 86.3 ÷ -5.01

= 17.23 cm

Hence, the focal length of the eyepiece is 17.23 cm

We simply divided the angular magnification magnitude from the focal length of objective lens so that the focal length of the eyepiece could come

A basketball is at rest on the floor. It is heated with a Bunsen burner to make it warmer. A student argues

that the thermal energy in the basketball could be converted to kinetic energy and the basketball will

spontaneously move upwards. This claim violates which law of nature?

Answers

Answer:

Newton's first law of motion - The law of inertia

Explanation:

The student's claim violates Newton's first law of motion which states that a body will continue in its state of rest or uniform motion except a force is applied to change that state of rest or uniform motion.

In the case of the student, no external force is applied to the basketball, so as to change its inertia, so it continues to stay in the same position. But, the student assumes that the kinetic energy of the ball will change as its thermal energy can be converted to kinetic energy.

This is incorrect because, the thermal energy only raises the temperature of the basketball and thus the energy of the molecules of the ball, it cannot cause the kinetic energy of the basketball to change since it applies NO FORCE to be basketball so as to change its inertia.

Which explanation best describes a wave?
A.
a disturbance that travels through a medium with a transfer of energy and matter
B.
a disturbance that travels through a medium without a transfer of energy and matter
C.
a disturbance that travels through a medium with a transfer of energy and without a transfer of matter
D.
a disturbance that travels through a medium with a transfer of matter and without a transfer of energy
E.
a disturbance that travels only in the absence of a material medium

Answers

Answer:

D

Explanation:

A disturbance that travels through a medium with a transfer of matter and without a transfer of energy best describes a wave. Option D  best describes a wave.

What is wave?

A wave is a phenomenon that flows across a material medium without leaving any lasting mark. Mechanical or electromagnetic waves can be used to classify it. Transverse and longitudinal are the two main forms.

Wave motion that is parallel to the wave direction is described as longitudinal wave.

A wave is a disturbance that moves across a medium with a transfer of matter , but not an energy transfer.

A disturbance that travels through a medium with a transfer of matter and without a transfer of energy best describes a wave.

Hence, option D  best describes a wave

To learn more about the wave, refer to the link;

brainly.com/question/3004869

#SPJ2

PLEASE HELP
A problem says a plane is accelerating
3.42 m/s2 northeast. Which one of these
tables includes that information correctly?

Answers

Answer:

The answer is C, I just guessed and got it right lol

Explanation:

10 You are given the following list of vo-
lumes: 0.45 m3; 375 cm3; 75 cL; 0.6 dm3.
a. Express all the volumes of the list in
liters.
b. Arrange these volumes in a descending
order.

Answers

Volumes can be transfered to liters only when their measurement units are dm³. a) Which means that: 0.45m³=450dm³ you move the decimal place three places to the right because m³ is larger measurement unit than dm³. 375 cm³ = 0.375 dm³; 75cl = 0.75l because of the prefix centi. 0.6dm³ = 0.6 dm³

So it will be 450l; 0.375l; 0.75l; 0.6l

b) 450l>0.75l>0.6l>0.375l

0.45m³>75cl>0.6dm³>375cm³

I hope i didn't make a mistake, but it would be good if somebody else answered too, so you can compare the answers and see if i made a mistake

The given volumes in liter will be 450 L, 0.375 L, 0.75 L, 0.6 L and the arrangement in descending order will be 450 L > 0.75 L > 0.6 L > 0.37 L .

What is Volume?

Every three-dimensional item takes up space in some way. The volume of this area is what is used to describe it. The area covered within an entity's three-dimensional bounds is referred to as its volume. The object's size is another name for it.

As per the given information in the question,

a.

The given volume in a meter cube is 0.45.

Unit conversion:

1 m³ = 1000 L

So, 0.45 m³ = 0.45 × 1000

= 450 L

The given volume in a centimeter cube is 375.

Unit conversion:

1 cm³ = 0.001 L

375 cm³ = 0.001 × 375

= 0.375 L.

The given volume in a centiliter cube is 75.

Unit conversion:

1 cl³ = 0.01 L

75 cl³ = 75 × 100

= 0.75 L

The given volume in the decimeter cube is 0.6.

1 dm³ = 1 L

0.6 dm³  = 0.6 L

b.

So, the volumes in descending order will be,

450 L > 0.75 L > 0.6 L > 0.37 L or,

0.45 m³ > 75 cl³ > 0.6 dm³ > 375 cm³

To know more about Volume:

https://brainly.com/question/13338592

#SPJ2

When carbon bonds with oxygen,
what
gas is formed?
When carbon bonds with oxygen, what gas is formed?

Answers

Answer:

Carbon dioxide

Explination:

I remember it from biology.

I hope this helps ^-^

In a chemical reaction, carbon combines with oxygen to form carbon dioxide gas.

What is a chemical reaction?

A chemical reaction can be defined as a process that causes the chemical transformation of one chemical substance to another. The chemical reactions accompany chemical changes that involve the rearrangement of electrons in the forming and breaking of bonds between atoms, with no change to the nuclei.

The substances which are primarily involved in a reaction are known as reactants or reagents. Chemical reactions are commonly characterized by a chemical change, and yield products, which exhibit properties different from the reactants.

Chemical reactions take place at a characteristic rate of reaction at a given temperature, pressure, and chemical concentration. The rates increase with the increasing temperature of the reaction when more thermal energy is available to achieve the activation energy for breaking bonds between atoms.

The combustion reaction takes place when carbon combines with oxygen to produce carbon dioxide gas.

Learn more about Chemical reactions, here:

https://brainly.com/question/22817140

#SPJ2

What is quantum physics? ​

Answers

Answer:

A fundamental theory that provides a description of the physical properties of nature at the scale of atoms and subatomic particles.

Explanation:

How is the L-section filter useful at both low and high frequencies?​

Answers

Answer: High pass filters, and in particular LC high pass filters are used in many RF applications where they block the lower frequencies and allow through higher frequency signals. Typically LC filters are used for the higher radio frequencies where active filters are not so manageable, and inductors more appropriate.

Answer:

high pass filters and in particular LC high pass filter are used in many RF application where they block the lower frequencies and allow through higher frequency signals. Typically LC filters are used for the higher radio frequencies where active filters are not so manageable and indicators more appropriate

We can see stars like the Sun because they​

Answers

Answer:

are nearby and bright i hope this helps

11.
How can we increase pressure?
Reducing surface area
Increasing surface area
Reducing force

Answers

Reducing Surface Area.

If the surface area becomes smaller, the pressure becomes larger.

You have probably noticed that carrying a person in a pool of water is much easier than carrying a person through air. To understand why, find the buoyant force exerted by air and by water on the person. Assume the average volume of a person is 0.092 m3, and that the person is submerged in air and water respectively. (Give the magnitude.)

air: ________
water :_______

Answers

Answer:

The answer is "0.91238 and 744.8"

Explanation:

In this scenario it is easier to take a person to the water-pool than to transport the people in the air, as the person's strength is increased by water upwards:

[tex]f_b \to m \to mg =person \\\\F_B \ in\ air = v\ & air\ g \\\\[/tex]

               [tex]=0.076 \times 1.225 \times 9.8 \\\\ =0.91238 \ N\\\\[/tex]

[tex]F_B \ in \ water = v \& water \ g \\\\[/tex]

                    [tex]=0.076 \times 1000 \times 9.8\\\\= 744.8 \ N\\[/tex]

An electron with an initial speed of 700,000 m/sm/s is brought to rest by an electric field. Did the electron move into a region of higher potential or lower potential?

Answers

Answer:

lower potential

Explanation:

Given that,

An electron with an initial speed of 700,000 m/s is brought to rest by an electric field. The electric field is defined as the electric force per unit charge. The electron must go in the opposite direction to get to the rest. It would imply that the electron would move in a region of lower potential.

Which statement is true for two pieces of iron at the same temperature ? A. The total kinetic energy of their particles is equal

Answers

The complete options are;

A. The average kinetic energy of their particles is the same.

B. The total kinetic energy of their particles is equal.

C. Heat flows from the larger object to the smaller object.

D. Heat flows from the object with higher potential energy to the object with lower potential energy.

Answer:

Explanation:

From the relationship between average kinetic energy and temperature, we have the formula;

E_k = (3/2)kT

Where;

k is a constant known as boltzmann constant.

T is known as temperature

We can see that at the same temperature (T), kinetic energy will remain the same because from the formula, E_k depends km only the temperature.

Thus, average kinetic energy of their particles saying that.

According to Newton, which of these will determine if and how an object will move?

1. how much energy the object has
2. how unstable the object is
3. whether a net force acts on the object
4. he total mass of the object

Answers

Answer:

3

Explanation:

Answer:

whether a net force acts on the object

o reduce the drag coefficient and thus to improve the fuel efficiency, the frontal area of a car is to be reduced. Determine the amount of fuel and money saved per year as a result of reducing the frontal area from 18 to 14 ft2. Assume the car is driven 12,000 mi a year at an average speed of 55 mi/h. Take the density and price of gasoline to be 50 lbm/ft3 and $3.10/gal, respectively; the density of air to be 0.075 lbm/ft3, the heating value of gasoline to be 20,000 Btu/lbm; and the overall efficiency of the engine to be 30 percent. Take the drag coefficient as CD

Answers

Complete question

To reduce the drag coefficient and thus to improve the fuel efficiency, the frontal area of a car is to be reduced. Determine the amount of fuel and money saved per year as a result of reducing the frontal area from 18 to 14 ft2. Assume the car is driven 12,000 mi a year at an average speed of 55 mi/h. Take the density and price of gasoline to be 50 lbm/ft3 and $3.10/gal, respectively; the density of air to be 0.075 lbm/ft3, the heating value of gasoline to be 20,000 Btu/lbm; and the overall efficiency of the engine to be 30 percent. Take the drag coefficient as CD=0.3 for a passenger car.

Answer:

22.22%

$57

Explanation:

From the question we are told that

Initial area of frontal area [tex]a_1=18ft^2[/tex]

Final area of frontal area [tex]a_2=14ft^2[/tex]

Distance covered a year [tex]D=12000mile[/tex]

Average speed a year [tex]V_{avg}=55mile/h[/tex]

Density [tex]\rho=50 lbm/ft3[/tex]

Price [tex]P= $3.10/gal[/tex]

Density of air [tex]\rho_{air} 0.075 lbm/ft3[/tex]

Heating value of gasoline [tex]Q=20,000 Btu/lbm[/tex]

Efficiency [tex]\eta=30\%[/tex]

Drag coefficient [tex]CD=0.3[/tex]

[tex]\triangle A=18-14ft^2=4ft^2[/tex]

Generally the equation for drag force [tex]C_D[/tex] is mathematically given as

[tex]F_D=\frac{C_DAPV^2}{2}[/tex]

[tex]F_D=\frac{0.3*18*0.075*(80.685^2)*A}{2}[/tex]

[tex]F_D=73.24Alb[/tex]

where [tex]v=55mil/h*1.467=80.685ft/s[/tex]

Generally the equation for work done W is mathematically given as[tex]W=F_D*L[/tex]

where [tex]L=12000mile*5280[/tex]

[tex]L=63360000[/tex]

[tex]W=73.24A*63360000[/tex]

[tex]W=4.6*10^9A[/tex]

Generally the equation for overall efficiency [tex]\eta[/tex] is mathematically given as

where

[tex]W_{req}=required\ gasoline\ power\ efficiency[/tex]

[tex]\eta=\frac{W}{W_{req}}[/tex]

[tex]W_{req}=\frac{W}{\eta}[/tex]

[tex]W_{req}=\frac{4.6*10^9}{0.3}[/tex]

[tex]W_{req}=1.55*10^{10}A[/tex]

Generally the equation for reduction fee with change in frontal area [tex]\triangle M[/tex] is mathematically given as

[tex]\triangle M =\triangle Vgasoline*cost[/tex]

Where

[tex]\triangle Vgasoline= volume\ reduction\ of\ gasoline[/tex]

[tex]\triangle Vgasoline=\frac{E_{req}}{H*P}[/tex]

[tex]\triangle Vgasoline=\frac{1.55*10^{10}A}{20000*778.169*32.2*50}[/tex]

if

[tex]20000btu/ibm=20000*778.169*32.2(1bm.ft^2/s)[/tex]

[tex]\triangle Vgasoline=\frac{1.55*10^{10}A}{20000*778.169*32.2*50}[/tex]

[tex]\triangle Vgasoline=0.61859A[/tex]

Therefore

[tex]\triangle M =0.61859A*3.10[/tex]

if [tex]1ft^2=7.48gal[/tex]

[tex]\triangle M =0.61859(4)*3.10*7.48[/tex]

[tex]\triangle M = \$ 57.671[/tex]

Generally the equation for reduction of fuel [tex]F_r[/tex]is mathematically given as

[tex]F_r=\frac{\triangle A}{\triangle i}*100[/tex]

where

[tex]\triangle i=18ft^2[/tex]

[tex]F_r=\frac{4}{18}*100[/tex]

Fuel reduction price by reducing front area is 22.22%

Money saved per year is $57

I've got this question, which option is correct?

Answers

Answer:

option c is the correct one

Why is Alternating Current important

Answers

It’s voltage can easily be modified and it allows power to be transmitted at a high voltage rate before being lowered to a smaller voltage rate to be safe.

Explanation:

La corriente alterna es de gran importancia, entre otras cosas, porque nos proporciona la red eléctrica domiciliaria. Es aquella con la cual funcionan habitualmente los transformadores y un gran número de dispositivos.

Other Questions
a sunglasses store bought $5,000 worth of glasses the store made $9,000 making a profit of $20 per sunglasses there were How many pairs of sunglasses involved? There are two spinners containing only white and green slices. Spinner A has 4 white slices and 1 green slice. All the slices are the same size. Spinner B has 3 white slices and 9 green slices. All the slices are the same size. Each spinner is spun. List these events from least likely to most likely. Event : Spinner B lands on a green slice. Event : Spinner A lands on a white or green slice. Event : Spinner A lands on a white slice. Event : Spinner B lands on a yellow slice. Anatom of an element has no electron,will that atom have any mass or not? Can antom existwithout electron? if so give an example. Reply fast this is due in some hours plz help with this question I got nothing but harassed today- comfort me PLEASE ANSWER!!!! Select the verb phrase and label it transitive or intransitive.Does Sam Spade give the police any information?transitive verb phrase intransitive verb phraseDoes Sam Spade give the police any information Does Sam Spade give the police any information What is the first step to solve this equation:11 - 3x = 44Add 3 to both sidesAdd 11 to both sidesSubtract 11 from both sidesDivide 3 on both sides. Given the reaction below which of the following best describes the energy changes in this reaction? Cu + Cl2 + energy > CuCl2 who win in a fight?6ix9ine or meek mill The volume of the polyhedron is ______ yd3. hi can you help me with my language arts ummmm i really need help Name two tips given in the passage for dealing with peer pressure. A 10 and a 15 resistor are connected in series across a 110V potential difference. (Can you find them) please help A) what is the total resistance of the circuit?B) what is the current through each resistor?C) what is the voltage drop across each resistor help please ! thank u sm How to solve for (3v^3r^4)^2 Dad is getting food ready for my birthday party! Including me, there will be 14 people at the party. Dad is baking cupcakes for us! He can bake 16 cupcakes per batch.How many batches should I tell Dad to make so we can each have 5 cupcakes? Ashley flies a drone 2,530 ft in 5 s. At this rate how many miles does the drone fly in a minute? 1 mi = 5280 ft Which type of epithelial tissue is best designed for gas exchange?